1989 AIME Problems/Problem 9
Contents
Problem
One of Euler's conjectures was disproved in the 1960s by three American mathematicians when they showed there was a positive integer such that ![\[133^5+110^5+84^5+27^5=n^{5}.\]](http://latex.artofproblemsolving.com/1/4/3/143026fa63a7ae20a66ec8c720201f4b53302ba9.png) Find the value of
 Find the value of  .
.
Solution 1 (FLT, CRT, Inequalities)
Taking the given equation modulo  and
 and  respectively, we have
 respectively, we have
 By either Fermat's Little Theorem (FLT) or inspection, we get
By either Fermat's Little Theorem (FLT) or inspection, we get
 By either the Chinese Remainder Theorem (CRT) or inspection, we get
By either the Chinese Remainder Theorem (CRT) or inspection, we get  
It is clear that  so the possible values for
 so the possible values for  are
 are  Note that
 Note that
 from which
from which  
If  then
 then
 which arrives at a contradiction. Therefore, we conclude that
which arrives at a contradiction. Therefore, we conclude that  
~MRENTHUSIASM
Solution 2
Note that  is even, since the LHS consists of two odd and two even numbers. By Fermat's Little Theorem, we know
 is even, since the LHS consists of two odd and two even numbers. By Fermat's Little Theorem, we know  Hence,
 Hence, ![\[n\equiv3+0+4+2\equiv4\pmod{5}.\]](http://latex.artofproblemsolving.com/2/b/1/2b1df66c5f4fb136139934ac01e1f99816282b15.png) Continuing, we examine the equation modulo
Continuing, we examine the equation modulo  
 ![\[n\equiv1-1+0+0\equiv0\pmod{3}.\]](http://latex.artofproblemsolving.com/e/4/c/e4c6715834530112be1c4e9f277a79fc6f63ce06.png) Thus,
Thus,  is divisible by three and leaves a remainder of four when divided by
 is divisible by three and leaves a remainder of four when divided by  It is obvious that
 It is obvious that  so the only possibilities are
 so the only possibilities are  or
 or  It quickly becomes apparent that
 It quickly becomes apparent that  is much too large, so
 is much too large, so  must be
 must be  
~Azjps (Solution)
~MRENTHUSIASM (Reformatting)
Solution 3
We can cheat a little bit and approximate, since we are dealing with such large numbers. As above,  and it is easy to see that
 and it is easy to see that  Therefore,
 Therefore,  so the last digit of
 so the last digit of  is
 is  
We notice that  and
 and  are all very close or equal to multiples of
 are all very close or equal to multiples of  We can rewrite
 We can rewrite  as approximately equal to
 as approximately equal to  This means
 This means  must be close to
 must be close to  
Note that  will obviously be too small, so we try
 will obviously be too small, so we try  and get
 and get  Bashing through the division, we find that
 Bashing through the division, we find that  which is very close to
 which is very close to  It is clear that
 It is clear that  will not give any closer of an answer, given the rate that fifth powers grow, so we can safely assume that
 will not give any closer of an answer, given the rate that fifth powers grow, so we can safely assume that  is the answer.
 is the answer.
Solution 4
In this solution we take advantage of the large numbers and utilize parity properties to give us a very good guess at the answer. The units digits of  are
 are  respectively, so the units digit of
 respectively, so the units digit of  is
 is  This tells us
 This tells us  is even. Since we are dealing with enormous numbers,
 is even. Since we are dealing with enormous numbers,  should not be that far from
 should not be that far from  Note that
 Note that  's units digit is
's units digit is  or
 or  When to the power of
 When to the power of  they each give
 they each give  and
 and  as the units digits. This further clues us that
 as the units digits. This further clues us that  ends in
 ends in  
 
Clearly,  so we start with
 so we start with  Now we need a way of distinguishing between numbers with units digit
 Now we need a way of distinguishing between numbers with units digit  We can do this by finding the last three digits for each of
 We can do this by finding the last three digits for each of  and
 and  which is not that difficult. For
 which is not that difficult. For  we have
 we have
 By the same reasoning, we get
By the same reasoning, we get
 Note that
Note that
 By observations,
By observations,  is obviously an overestimate. So, the answer is
 is obviously an overestimate. So, the answer is  
~jackshi2006 (Solution)
~MRENTHUSIASM (Revisions and  Adjustments)
 Adjustments)
Solution 5
First, we take mod  on both sides to get
 on both sides to get  . Mod
. Mod  gives
 gives  . Also, mod
. Also, mod  gives
 gives  (by FLT). Finally, note that mod
 (by FLT). Finally, note that mod  gives
 gives  . Thus,
. Thus,
 By CRT,
By CRT,  , so
, so  is one of
 is one of  . However,
. However,  , so
, so  . Thus,
. Thus,  .
.
~brainfertilzer
Solution 6 (Brute Force)
We have ![\[n^5 = 133^5 + 110^5 + 84^5 +27^5 = 61917364224,\]](http://latex.artofproblemsolving.com/8/9/5/895c0caa4b5c659a02ab26c6f0c72b972fab20b1.png) for which
 for which ![$n = \sqrt [5]{61917364224} = \boxed{144}.$](http://latex.artofproblemsolving.com/6/a/7/6a7b7e25c2a218baf4e841d289b45116e1b22fcc.png) 
Solution 7 (Cheese)
By approximation,  ,
,  , and
, and  . Adding the fifth power of each fraction of
. Adding the fifth power of each fraction of  gives a total of roughly
 gives a total of roughly  . By testing, we know that the fifth root of this value is less than
. By testing, we know that the fifth root of this value is less than  , as
, as  , meaning we have reduced
, meaning we have reduced  to
 to  .
The last digit of all four numbers, in order, are
.
The last digit of all four numbers, in order, are  ,
,  ,
,  , and
, and  , which is gained through writing out the cycles of the last digit up to the fifth power. This means that the answer has a units digit of
, which is gained through writing out the cycles of the last digit up to the fifth power. This means that the answer has a units digit of  , reducing
, reducing  down to either
 down to either  or
 or  . Since the previous cheesing shows obviously that the value is closer to
. Since the previous cheesing shows obviously that the value is closer to  than it is to
 than it is to  , we know our final answer of
, we know our final answer of  is correct.
 is correct.
Cheese solution by juwushu.
Solution 8 (Guesswork or Bash )
We know that  for all a. You can prove this later. So we know the number ends with 4. Since 133 is the biggest number, while everything else is small, we can safely assume that it is
 for all a. You can prove this later. So we know the number ends with 4. Since 133 is the biggest number, while everything else is small, we can safely assume that it is  , because 5th powers vary wildly. It is not 134 because
, because 5th powers vary wildly. It is not 134 because  plays a role We can also just take
 plays a role We can also just take  and the other numbers
 and the other numbers  , add them up, and we realize that
, add them up, and we realize that  is close to this. Then we also get
 is close to this. Then we also get  
 
Remark
We could also take this expression mod  which would give
 which would give  Then the only reasonable number that works is
 Then the only reasonable number that works is  so we could safely assume this is the answer.
 so we could safely assume this is the answer.
See also
| 1989 AIME (Problems • Answer Key • Resources) | ||
| Preceded by Problem 8 | Followed by Problem 10 | |
| 1 • 2 • 3 • 4 • 5 • 6 • 7 • 8 • 9 • 10 • 11 • 12 • 13 • 14 • 15 | ||
| All AIME Problems and Solutions | ||
These problems are copyrighted © by the Mathematical Association of America, as part of the American Mathematics Competitions.  
